連立方程式(解の存在条件)

実数\(m\)を定数とする.\(x\)と\(y\)に関する連立1次方程式\(\begin{cases}2x+y-2=0\\mx-y-3m+1=0\end{cases}\)が\(x>0\)かつ\(y>0\)である解をもつための必要十分条件を求めよ.

(慶応大)

受験勉強を始めると最初の方で出会ってやる気を削ぐ系の問題^^;

解答

\begin{align*}
&~\exists x \exists y \left[\begin{cases}2x+y-2=0\\mx-y-3m+1=0\end{cases}\land x>0 \land y>0\right]\\
&~\exists x \exists y \left[\begin{cases}2x+y-2=0\\(m+2)x-3m-1=0\end{cases}\land x>0 \land y>0\right]\\
\Longleftrightarrow&~\exists x \exists y \left[2x+y-2=0 \land (m+2)x-3m-1=0 \land x>0 \land y>0 \right. \\
&\left.\land (m+2=0 \lor m+2 \neq 0)\right]\\
\Longleftrightarrow&~\exists x \exists y \left[(2x+y-2=0 \land (m+2)x-3m-1=0 \land x>0 \land y>0 \land m+2=0 )\right.\\
&\lor \left.(2x+y-2=0 \land (m+2)x-3m-1=0 \land x>0 \land y>0 \land m+2 \neq 0)\right]\\
\Longleftrightarrow&~\exists x \exists y \left[(2x+y-2=0 \land 5=0 \land x>0 \land y>0 \land m=-2) \right.\\
&\lor \left.(2x+y-2=0 \land (m+2)x-3m-1=0 \land x>0 \land y>0 \land m \neq -2)\right]\\
\Longleftrightarrow&~\exists x \exists y \left[y=-2x+2 \land x =\frac{3m+1}{m+2} \land x>0 \land y>0 \land m \neq -2)\right]\\
\Longleftrightarrow&~\exists x \exists y \left[y=-2\frac{3m+1}{m+2}+2 \land x =\frac{3m+1}{m+2} \land x>0 \land y>0 \land m \neq -2)\right]\\
\Longleftrightarrow&~\exists x \exists y \left[y=\frac{2-4m}{m+2} \land x =\frac{3m+1}{m+2} \land x>0 \land y>0 \land m \neq -2)\right]\\
\Longleftrightarrow&~\frac{2-4m}{m+2}>0 \land \frac{3m+1}{m+2}>0 \land m \neq -2\\
\Longleftrightarrow&~\frac{2-4m}{m+2}>0 \land \frac{3m+1}{m+2}>0 \land (m+2>0 \lor m+2<0)\\ \Longleftrightarrow&~ \left( \frac{2-4m}{m+2}>0 \land \frac{3m+1}{m+2}>0 \land m+2>0 \right)\\
&\lor \left(\frac{2-4m}{m+2}>0 \land \frac{3m+1}{m+2}>0 \land m+2<0\right)\\ \Longleftrightarrow&~ \left( 2-4m>0 \land 3m+1>0 \land m>-2 \right)\\
&\lor \left(2-4m<0 \land 3m+1<0 \land m<-2\right)\\
\Longleftrightarrow&~ \left( m<\frac{1}{2} \land m>-\frac{1}{3} \land m>-2 \right)\lor \left(m>\frac{1}{2} \land m<-\frac{1}{3} \land m<-2\right)\\
\Longleftrightarrow&~ m<\frac{1}{2} \land m>-\frac{1}{3} \land m>-2\\
\Longleftrightarrow&~ -\frac{1}{3} < m < \frac{1}{2}
\end{align*}

解答終

こう考えればただの(論理)計算問題なのでめっちゃ楽。

値域の問題

実数\(a,b,c\)が\(a+b+c=a^2+b^2+c^2=1\)を満たすとする.\(c\)のとりうる値の範囲を求めよ.

「☆という値をとる」という主張を「その値☆を実現するような★が存在する」と言い換えます。あとはそれを機械的に処理するだけ。

解答

\begin{align*}
&\text{\(c\)が\(k\)という値をとる}\\
\Longleftrightarrow&~\exists a\in \mathbb{R}\exists b\in \mathbb{R}[a+b+k=a^2+b^2+k^2=1]\\
\Longleftrightarrow&~\exists a\in \mathbb{R}\exists b\in \mathbb{R}[a+b+k=1 \land a^2+b^2+k^2=1]\\
\Longleftrightarrow&~\exists a\in \mathbb{R}\exists b\in \mathbb{R}[b=1-a-k \land a^2+b^2+k^2=1]\\
\Longleftrightarrow&~\exists a\in \mathbb{R}[a^2+(1-a-k )^2+k^2=1]\\
\Longleftrightarrow&~\exists a\in \mathbb{R}[a^2-(1-k )a+k^2-k=0]\\
\Longleftrightarrow&~(1-k )^2-4(k^2-k)\geq 0\\
\Longleftrightarrow&~3k^2-2k-1\leq 0\\
\Longleftrightarrow&~-\frac{1}{3}\leq k \leq 1\\
\end{align*}

解答終

2次方程式の共通解問題(その2・つづき)

\[(P\Rightarrow Q \lor R) \land \overline{Q\Rightarrow P} \land (R \Rightarrow P)\Rightarrow~(P \Leftrightarrow R)\]

証明

\(P\Rightarrow Q \lor R,\overline{Q\Rightarrow P},R \Rightarrow P\)となる行,すなわち\(P\rightarrow Q \lor R,\overline{Q\rightarrow P},R \rightarrow P\)が真となる行(上から6行目)に着目すると,\((P\rightarrow Q \lor R) \land \overline{Q\rightarrow P} \land (R \rightarrow P)\)と\(P \leftrightarrow R\)の真理値(青〇)が一致している.したがって\[(P\Rightarrow Q \lor R) \land \overline{Q\Rightarrow P} \land (R \Rightarrow P)\Longrightarrow~(P \Leftrightarrow R)\]を得る.

証明終

(関連:2次方程式の共通解問題(その2)

2次方程式の共通解問題(その2)

\(2\)つの\(2\)次方程式\(x^2-3x+m-1=0,x^2+(m-2)x-2=0\)が共通な実数解をただ\(\)1つもつとき,定数\(m\)の値とその共通解を求めよ.

解答

\begin{align*}
&x^2-3x+m-1=0,x^2+(m-2)x-2=0\text{が共通な実数解をただ1つもつ}\\
\Longrightarrow~&x^2-3x+m-1=0,x^2+(m-2)x-2=0\text{が共通な実数解をもつ}\\
\Longleftrightarrow~&\exists x
\begin{cases}
x^2-3x+m-1=0\\
x^2+(m-2)x-2=0
\end{cases}\\
\Longleftrightarrow~&\exists x
\begin{cases}
x^2-3x+m-1=0\\
x^2+(m-2)x-2-(x^2-3x+m-1)=0
\end{cases}\\
\Longleftrightarrow~&\exists x
\begin{cases}
x^2-3x+m-1=0\\
(m+1)(x-1)=0\end{cases}\\
\Longleftrightarrow~&\exists x
\begin{cases}
x^2-3x+m-1=0\\
m=-1\lor x=1
\end{cases}\\
\Longleftrightarrow~&\exists x [x^2-3x+m-1=0 \land (m=-1 \lor x=1)]\\
\Longleftrightarrow~&\exists x [(x^2-3x+m-1=0 \land m=-1) \lor (x^2-3x+m-1=0 \land x=1)]\\
\Longleftrightarrow~&\exists x (x^2-3x-2=0 \land m=-1) \lor \exists x(1^2-3\cdot 1+m-1=0 \land x=1)]&\\
\Longleftrightarrow~&\exists x \left[x=\frac{3\pm \sqrt{17}}{2} \land m=-1\right] \lor \exists x[m=3 \land x=1]\\
\Longleftrightarrow~&\left(\exists x \left[x=\frac{3\pm \sqrt{17}}{2}\right] \land m=-1 \right) \lor (m=3 \land \exists x[x=1])\\
\Longleftrightarrow~&m=-1 \lor m=3
\end{align*}
(\(\exists x \left[x=\frac{3\pm \sqrt{17}}{2}\right],\exists x[x=1]\)は恒真命題)二行目が同値変形でないことに注意すると,結局,
\begin{align*}
x^2-3x+m-1=0,x^2+(m-2)x-2=0\text{が共通な実数解をただ1つもつ}&\\
\Longrightarrow~m=-1 \lor m=3&
\end{align*}
つまり得られた条件は必要条件に過ぎないので,十分性を調べる必要があります。そこで,逆に\(m=-1\)のときと\(m=3\)のときそれぞれの場合において「与えられた\(2\)次方程式が共通な実数解をただ1つもつ」ことを調べることにします。

\(m=-1\)のとき,与えられた\(2\)つの方程式は\(x^2-3x-2=0\)と\(x^2-3x-2=0\)となり,どちらの解も\(x=\frac{3\pm\sqrt{17}}{2}\)であり「ただ\(1\)つの」共通解を持つとは当然いえません。他方,\(m=3\)のときは\(x^2-3x+2=0\)と\(x^2+x-2=0\)となり,これらを解くとそれぞれの解は\(x=-2\)と\(x=-1\),そして\(x=-2\)と\(x=1\)となりこれなら「ただ\(1\)つの」共通解\(x=-2\)をもつと言えます。したがって答えは\[m=3\](で,共通解は\(x=-2\))となります。

解答終

結局これは,①必要条件を調べ,次に②その条件が十分条件となっているかどうかを調べる,という2つの段階に分けるというのが大まかなシナリオです(①は同値性を気にすることなくとりあえず右向きの矢印だけ気にすればいいから気楽)。このような方針は問題を解く際にしばしば見られるものです。実際,教科書の軌跡の解説などではお馴染みですね。僕は受験生時代,この問題の解説にはどことない気持ち悪さを感じつつもただただ解法パターンとして覚えることしかできず,細かいことは見て見ぬふりをしていました。今思えばその「気持ち悪さ」は結局論理を理解していなかったのが原因だと思います。とはいえ,学校で扱わないのだからこの手の話が分からんのはアタリマエ。ってかそもそも扱ってないことを問題にする時点でおかしくないか…?

先日,授業でこの\(2\)次方程式の共通解問題を扱い,例によって上のような解説を(もちろん論理式でなく日本語で)していてふと思いました。上の議論はつまり「\(P\Longrightarrow Q \lor R\)が言えました,そして\(Q\rightarrow P\)が偽(\(\overline{Q\rightarrow P}\)が真)で,\(R \rightarrow P\)が真であることが分かりました,だから\(P \Longleftrightarrow R\)と言えるよね」というもの,つまり\[(P\Rightarrow Q \lor R) \land \overline{Q\Rightarrow P} \land (R \Rightarrow P)\Longrightarrow~(P \Leftrightarrow R)\]ですが(※),そもそもこの命題は正しいのでしょうか…?僕自身この変形を普段から無意識に行っていましたが…よくよく考えれば疑問です。このことを調べてみます。(つづく

(関連:2次方程式の共通解問題(その1)

※(21/9/16) \(\Leftrightarrow\)を\(\Rightarrow\)に訂正しました。R君ご指摘ありがとうございます。

かつとまたは,どっち?

不等式\[4x^2+5x-12\leq 3|x|\]を解け.

解答1

\((\mathrm{i})~\)\(x \geq 0\)のとき\(|x|=x\)であるから,
\begin{align*}
&4x^2+5x-12\leq 3x\\
&4x^2+2x-12\leq 0\\
&2x^2+x-6\leq 0\\
&(2x-3)(x+2)\leq 0\\
&-2\leq x \leq \frac{3}{2}\\
\end{align*}\(x \geq 0\)との共通部分を考え,\(0 \leq x \leq \frac{3}{2}\tag{1}\)

\((\mathrm{ii})~\)\(x < 0\)のとき\(|x|=-x\)であるから,
\begin{align*}
&4x^2+5x-12\leq -3x\\
&4x^2+8x-12\leq 0\\
&x^2+2x-3\leq 0\\
&(x+3)(x-1)\leq 0\\
&-3\leq x \leq 1\\
\end{align*}\(x < 0\)との共通部分を考え,\(-3 \leq x < 0\tag{2}\)

\((1),(2)\)との和集合を考え,\(-3 \leq x \leq \frac{3}{2}\)

解答終

よく見るいたって普通の解答ですが,生徒に「なぜ\((\mathrm{i}),(\mathrm{ii})\)では『かつ』なのに最後は『または』なんですか?『かつ』じゃだめなんですか?」と問われたらなんと答えらたいいのだろう?教科書にはもちろんそれらしい記述は一切はないから例によって頼りにならない。説明がないのだから,結局「そういうものだから覚えろ」と言うかそもそも(生徒の感覚に任せ)触れないかのどちらかになると思う。

解答2

\begin{align*}
&4x^2+5x-12\leq 3|x|\\
\Longleftrightarrow~&4x^2+5x-12\leq 3|x| \land ( x \geq 0 \lor x < 0)\\ \Longleftrightarrow~&(4x^2+5x-12\leq 3|x| \land x \geq 0) \lor (4x^2+5x-12\leq 3|x| \land x < 0)\\ \Longleftrightarrow~&(4x^2+5x-12\leq 3x \land x \geq 0) \lor (4x^2+5x-12\leq -3x \land x < 0)\\ \Longleftrightarrow~&(2x^2+x-6 \leq 0 \land x \geq 0) \lor (x^2+2x-3\leq 0 \land x < 0)\\ \Longleftrightarrow~&( -2\leq x \leq \frac{3}{2} \land x \geq 0) \lor (-3 \leq x \leq 1 \land x < 0)\\ \Longleftrightarrow~& 0 \leq x \leq \frac{3}{2}\lor -3 \leq x < 0\\ \Longleftrightarrow~&-3 \leq x \leq \frac{3}{2} \end{align*} 解答終

結局,やっていることが,恒真命題の追加と分配法則と分かり,これなら先のような疑問を挟む余地がありません。

\(*\)\(*\)\(*\)

教科書ってこの種のその先はいう必要ないですよね的な部分があるからいまいち信用できないし,またそれを妄信する人も理解できない。教科書の解答こそが理想の解答だとかほんと勘弁。

2次方程式が異符号の解をもつための条件

チャート式なんかによくみる解法

\(2\)次方程式\(f(x)=0\)が異なる\(2\)つの正の解をもつ\(~\Longleftrightarrow D>0,\alpha+\beta >0,\alpha\beta > 0\)
\(2\)次方程式\(f(x)=0\)が異なる\(2\)つの負の解をもつ\(~\Longleftrightarrow D>0,\alpha+\beta <0,\alpha\beta < 0\)
\(2\)次方程式\(f(x)=0\)が異符号の解をもつ\(~\Longleftrightarrow \alpha\beta < 0 \)

これを見てふと思う。最後のだけなんで\(D>0\)がないの?と。補足をよく見ると「このとき,\(D>0\)は成り立っている」と小さく書いてある。ここ,ちょっと疑問をもちつつも「まあそういうもんなんだろ~」程度のゆるい理解で済ませているひとも少なくないと思います。でも,ここをちゃんと確認しないまま結果だけ使うというのは,理由を納得しないまま覚えるということで,数学としてはその姿勢はちょっと不安です。

証明してみます。以下,\(\alpha,\beta\)を\(2\)次方程式\(f(x)=0\)の解とします。

証明

\(2\)次方程式\(f(x)=0\)が異符号の解をもつとする.
\begin{align*}
&~\text{\(2\)次不等式\(f(x)=0\)が異符号の解をもつ}\\
\Longleftrightarrow &~D>0 \land ((\alpha > 0 \land \beta < 0) \lor (\alpha < 0 \land \beta > 0))\\
\Longleftrightarrow &~D>0 \land \alpha\beta < 0\\
\Longrightarrow &~\alpha\beta < 0 \quad\text{※ 必要条件}
\end{align*}よって,\[\text{\(2\)次不等式\(f(x)=0\)が異符号の解をもつ}\Longrightarrow ~\alpha\beta < 0\]が成り立つ.逆に,\(\alpha\beta < 0\)とする.
\begin{align*}
\alpha\beta=\frac{c}{a}<0 \Longleftrightarrow~& ac<0\\ \Longleftrightarrow~& -4ac > 0\\
\Longrightarrow~& b^2-4ac > 0 \quad\text{※ 必要条件}\\
\Longleftrightarrow~& D > 0
\end{align*}ゆえに,
\begin{align*}
\alpha\beta < 0 \Longrightarrow~&D>0 \land \alpha\beta < 0\\
\Longleftrightarrow~&\text{\(2\)次不等式\(f(x)=0\)が異符号の解をもつ}
\end{align*}以上により,\[\text{\(2\)次不等式\(f(x)=0\)が異符号の解をもつ}\Longleftrightarrow~\alpha\beta < 0\]が示された.

証明終

判別式と解と係数の関係どーのこーのがこの分野のテーマだと思うんですが,そんなことより補足として小さく書かれたこっちの議論の方が大事だし面白いと個人的に思います^^;

そもそも,教科書や教科書準拠問題集はそもそもこの辺の論理にはあまり深入りしない傾向がある気がします。卑近な例ですが例えば「判別式は\(D\)じゃなくて\(\frac{D}{4}\)を使うと計算が楽だよ!」とか。これだって,「判別式はあくまで\(D\)であって,\(\frac{D}{4}\)だなんて勝手に\(\frac{1}{4}\)しちゃだめだろ」と思いませんでしたか…?僕は思ったなあ。実際楽なので正当性も確かめずに使ってましたが。まあともかくこれだって,なんのことはない,\begin{align*}D > 0 \Longleftrightarrow \frac{D}{4}>0\\
D = 0 \Longleftrightarrow \frac{D}{4}=0\\
D < 0 \Longleftrightarrow \frac{D}{4} <0\\ \end{align*}ということに過ぎず,したがって例えば\(\frac{D}{4}>0\)を変形して得られる結論は,\(D>0\)と同値であるわけです。だから\(\frac{D}{4}>0\)で考えてよい,という。

もっとも,細かいことは気にせずとりあえず使えるように(=問題が解けて,点数がもらえるように)なることを目指し,その後改めて,細部を振り返り精査していく…という勉強法は難しい内容を学びとるためのひとつの有効な姿勢であり,決して否定はできません。あまり細かいことを言うと敷居が高くなったり(※誤用の方),あるいは深入りしすぎて手段と目的が逆転してしまったりといいことばかりではありませんしね。どう導入するのか,というのは難しいところです。

\(0=1\)の証明?

youtubeのおもしろ動画。これを不思議と思うか,あるいは一目でツッコミを入れられるかどうか,論理の勉強になる動画だと思います。

(\(0=1\)の証明?)

\begin{align}
-20 &=-20 \tag{1}\\
16-36 &= 25-45 \tag{2}\\
4^2-4\cdot 9 &= 5^2 – 5\cdot 9 \tag{3}\\
4^2-4\cdot 9 + \frac{81}{4} &= 5^2 – 5\cdot 9 + \frac{81}{4} \tag{4}\\
4^2-2 \cdot 4\cdot \frac{9}{2} + \left(\frac{9}{2}\right)^2 &= 5^2 – 2 \cdot 5\cdot \frac{9}{2} + \left(\frac{9}{2}\right)^2 \tag{5}\\
\left(4 – \frac{9}{2}\right)^2&= \left(5 – \frac{9}{2} \right)^2 \tag{6}\\
4 – \frac{9}{2}&= 5 – \frac{9}{2} \tag{7}\\
4 &= 5 \tag{8}\\
4 -4 &= 5 – 4 \tag{9}\\
0 &= 1 \tag{10}
\end{align}私たちは一般に計算問題が与えられば式を次々と改行・羅列して「答え」を求め,とにもかくにも「答え」さえ手に入れば,とくに疑問も抱くことなくそれを解答欄に書いてさっさと次の問題に進みがちです。しかし,細かいことをいえば本来はその各行の式と式の間には論理的にどういう関係があるのか,まで考える必要があります:
たとえば,\(-20 =-20\)と\(16-36 = 25-45 \)の間にはどんな関係があるでしょうか。\(16-36=-20\)そして\(25-45=-20\)という等式が成り立ちますから,\(-20 =-20 \)という仮定から\(16-36 = 25-45 \)が導けます。すなわち,\[-20 =-20 \Longrightarrow 16-36 = 25-45\]逆に,\(16-36 = 25-45 \)という仮定から\(-20 =-20 \)も確かに導けます。\[-20 =-20 \Longleftarrow 16-36 = 25-45\]この二つをあわせて,\[-20 =-20 \Longleftrightarrow 16-36 = 25-45\]と書きます。

式\((1)\)と式\((2)\)はいわば互いに‘行き来’できる関係(これを以後「同値」と呼ぶことにします)がありましたが,一般には必ずしもこのような関係が成り立つとは限りません(感覚的にたとえると「犬ならば哺乳類」ですが「哺乳類ならば犬」とは限りません)。この点に注意して,各行における「式と式の論理関係」を同様に調べていくと\((1)\)から\((6)\)までの式,また\((7)\)から\((10)\)までの式が互いに同値であることは一目でわかります。怪しいのは\((6)\)式と\((7)\)式の間の論理関係です。見た目がうるさいので,ここでは\(4 – \frac{9}{2} = a\),\(5 – \frac{9}{2} = b\)とおいて考えることにします。

\(a^2=b^2 \Leftarrow a=b\)は言えるか?
これは明らかに言えます。仮定\(a=b\)の両辺を\(2\)乗すればいいだけですから。

\(a^2=b^2 \Rightarrow a=b\)は言えるか?
これはいけない。なぜなら,\(a^2=b^2\)という仮定からは,\(a=b\)だけでなく,\(a=-b\)という可能性も考えられますから。実際,
\begin{align*}
&a^2=b^2\\
\Longrightarrow~&a^2-b^2=0\\
\Longrightarrow~&(a-b)(a+b)=0\\
\Longrightarrow~&a-b=0\text{または}a+b=0\\
\Longrightarrow~&a=b\text{または}a=-b
\end{align*}(※逆も成り立つ)

以下,正しい論理(式)を書いてみます(または,を\(\lor\)と書くことにします)。
\begin{align}
&-20 =-20 \\
\Longleftrightarrow &~16-36 = 25-45 \\
\Longleftrightarrow &~4^2-4\cdot 9 = 5^2 – 5\cdot 9 \\
\Longleftrightarrow &~4^2-4\cdot 9 + \frac{81}{4} = 5^2 – 5\cdot 9 + \frac{81}{4} \\
\Longleftrightarrow &~4^2-2 \cdot 4\cdot \frac{9}{2} + \left(\frac{9}{2}\right)^2 = 5^2 – 2 \cdot 5\cdot \frac{9}{2} + \left(\frac{9}{2}\right)^2 \\
\Longleftrightarrow &~\left(4 – \frac{9}{2}\right)^2= \left(5 – \frac{9}{2} \right)^2 \\
\Longleftrightarrow &~4 – \frac{9}{2}= 5 – \frac{9}{2} \lor 4 – \frac{9}{2}= – \left( 5 – \frac{9}{2}\right)\\
\Longleftrightarrow &~4 = 5 \lor 9 = 9 \\
\Longleftrightarrow &~9 = 9\\
\end{align}結局,\((1)\)から\((5)\)はいわば‘目くらまし’で,\(-20=-20\)という面白みのない仮定から,\(9=9\)というやはり面白みのない結論が得れらた,ということに過ぎない,ということでした。

文字は死んで変域残す

\(x,y\)が実数で,\(2x^2+3xy+2y^2=1\)を満たすとき,\(x+y+xy\)の最大値と最小値を求めよ.

という定番の問題についてみてみます.これは\(x+y=u,xy=v\)とおいたあと,「\(x,y\)が実数」という条件を\(u,v\)に反映させるのがポイントなのでした.すなわち,\(t^2-ut+v=0\)の判別式を\(\geq 0\)とすることにより
\[u^2-4v \geq 0\]
この不等式に注意しながら\(x+y+xy\)の最大値・最小値を調べる,という流れが定石でした(虎は死んで皮を残す,人は死んで名を残す,文字は死んで変域を残す…).この,\(u^2-4v \geq 0\)を得る流れは論理的にはどうなっているのか,調べてみます.

解答
\begin{align*}
&\text{\(x+y+xy\)が\(k\)という値をとる}\\
\Longleftrightarrow~&\exists x \exists y[x+y+xy=k \land 2x^2+3xy+2y^2=1]\\
\Longleftrightarrow~&\exists x \exists y \left[x+y+xy=k \land 2x^2+3xy+2y^2=1 \land \exists u \exists v \begin{cases}x+y=u\\xy=v\end{cases}\right]\\
\Longleftrightarrow~&\exists x \exists y \exists u \exists v\left[x+y+xy=k \land 2(x+y)^2-xy=1 \land \begin{cases}x+y=u\\xy=v\end{cases}\right]\\
\Longleftrightarrow~&\exists x \exists y \exists u \exists v\left[u+v=k \land 2u^2-v=1 \land \begin{cases}x+y=u\\xy=v\end{cases}\right]\\
\Longleftrightarrow~&\exists u \exists v\left[u+v=k \land 2u^2-v=1 \land \exists x \exists y \begin{cases}x+y=u\\xy=v\end{cases}\right]\tag{\(\ast\)}
\end{align*}
ここで
\begin{align*}
&\exists x \exists y \begin{cases}x+y=u\\xy=v\end{cases}\\
\Longleftrightarrow~&\exists x \exists y \begin{cases}x+y=u\\ \frac{1}{4}((x+y)^2-(x-y)^2)=v\end{cases}\\
\Longleftrightarrow~&\exists x \exists y \begin{cases}x+y=u \\ (x-y)^2=u^2-4v\end{cases}\\
\Longleftrightarrow~&\exists x \exists y \begin{cases}x+y=u \\ |x-y|=\sqrt{u^2-4v}\end{cases}\\
\Longleftrightarrow~&\exists x \exists y \begin{cases}y=u-x \\ (x-y=\sqrt{u^2-4v} \land x-y \geq 0 ) \lor (y-x=\sqrt{u^2-4v} \land x-y < 0 )\end{cases}\\ \Longleftrightarrow~&\exists x \left[\left(x=\frac{1}{2}\left(u+\sqrt{u^2-4v}\right) \land x \geq \frac{u}{2} \right) \lor \left(x=\frac{1}{2}\left(u-\sqrt{u^2-4v}\right) \land x < \frac{u}{2} \right)\right]\\ \Longleftrightarrow~&\frac{1}{2}\left(u+\sqrt{u^2-4v}\right) \geq \frac{u}{2} \lor \frac{1}{2}\left(u-\sqrt{u^2-4v} \right) < \frac{u}{2} \\ \Longleftrightarrow~&\sqrt{u^2-4v} \geq 0 \lor \sqrt{u^2-4v} < 0\\ \Longleftrightarrow~&\sqrt{u^2-4v} \geq 0\\ \Longleftrightarrow~&u^2-4v \geq 0\\ \end{align*} であるから
\begin{align*}
(\ast)\Longleftrightarrow~&\exists u \exists v\left[u+v=k \land v=2u^2-1 \land u^2-4v \geq 0\right]\\
\Longleftrightarrow~&\exists u \left[u+(2u^2-1)=k \land u^2-4(2u^2-1) \geq 0\right]\\
\Longleftrightarrow~&\exists u \left[k = 2u^2+u-1 \land -\frac{2}{\sqrt{7}} \leq u \leq \frac{2}{\sqrt{7}}\right]\\
\Longleftrightarrow~&-\frac{9}{8} \leq u \leq \frac{1}{7}+\frac{2}{\sqrt{7}}
\end{align*}
ゆえに,最大値\(\displaystyle \frac{1}{7}+\frac{2}{\sqrt{7}}\),最小値\(\displaystyle -\frac{9}{8}\).

\(\ast\)    \(\ast\)    \(\ast\)

ここで,~であるから」までがいわゆる「実数の存在条件」の処理です.前回の「\(m^2<\frac{1}{12}\)を満たす実数の存在条件は~」と同じ考え方で導出してみました.

◆無理不等式その2

次の式を\(\sqrt{\quad}\)のない形で表せ(同値変形せよ).
\[\sqrt{a} < b\]

恒真条件の追加と分配法則,矛盾命題の消去により,
\begin{align*}
&\sqrt{a} < b\\ \Longleftrightarrow~&\sqrt{a}< b \land (b \geq 0 \lor b < 0)\\ \Longleftrightarrow~&(\sqrt{a} < b \land b \geq 0)\lor (\sqrt{a} < b \land b < 0)\\ \Longleftrightarrow~&\sqrt{a} < b \land b \geq 0 \end{align*} ここからさらに変形を考えますが,前回同様,いきなり同値な変形は考えづらいので,必要性\((\Rightarrow)\)と十分性\((\Leftarrow)\)を別々に考えることにします. まず必要性\((\Rightarrow)\)から.\(\sqrt{a} \geq 0\)ですから,\(\sqrt{a} < b\)の両辺を2乗することができて,例えば次のように必要条件が得られます: \begin{align*} &\sqrt{a} < b \land b \geq 0 \Longrightarrow a < b^2 \land b \geq 0 \tag{1} \end{align*} 次にこの\((1)\)における十分性\((\Leftarrow)\)を考えてみます.当然,\(a < b^2\)の両辺に\(\sqrt{\quad}\)をとりたくなりますが,しかし\(a\)が正である保証は今手元の仮定にはありません.つまり\(\sqrt{\quad}\)をとることができず,戻れない.そこで,\((1)\)において必要条件をもう少し絞り出すことを考えます.欲しいのは\(a \geq 0\)ですが,\(\sqrt{a}\)の‘中身’は正ですから,必要条件は \[\sqrt{a} < b\land b \geq 0 \Longrightarrow a < b^2 \land b \geq 0 \land a \geq 0\] とできるはずです.そして改めて十分性を確認してみます. \begin{align*} a < b^2 \land b \geq 0 \land a \geq 0 \Longrightarrow &\sqrt{a} < \sqrt{b^2} \land b\geq 0 \land a \geq 0\\ \Longrightarrow &\sqrt{a} < |b| \land b \geq 0 \land a \geq 0\\ \Longrightarrow &\sqrt{a} < b \land b \geq 0 \land a \geq 0\\ \Longrightarrow &\sqrt{a} < b \land b \geq 0 \end{align*} となり戻れました.これで必要十分(同値)であることが分かりました.したがって\((1)\)の論理式は, \[\sqrt{a} < b \land b \geq 0 \Longleftrightarrow a < b^2 \land b \geq 0 \land a \geq 0 \Longleftrightarrow 0\leq a < b^2 \land b \geq 0 \] と書きかえれば同値になることが分かりました. 以上により,

\[\sqrt{a} < b \Longleftrightarrow 0\leq a < b^2 \land b \geq 0 \]

と同値変形できることが分かりました.

◆無理不等式その1

次の式を\(\sqrt{\quad}\)のない形で表せ(同値変形せよ).
\[\sqrt{a}>b\]

恒真条件の追加と分配法則により,
\begin{align*}
&\sqrt{a}>b\\
\Longleftrightarrow~&\sqrt{a}>b \land (b \geq 0 \lor b < 0)\\
\Longleftrightarrow~& (\sqrt{a}>b \land b \geq 0)\text{(ア)} \lor (\sqrt{a}>b \land b < 0) \text{(イ)}
\end{align*}

(ア)と(イ)を分けて考えます.

(まず(ア)について)
いきなり同値な変形は考えづらいので,必要性\((\Rightarrow)\)と十分性\((\Leftarrow)\)を別々に考えることにします. まず必要性\((\Rightarrow)\)から.今,\(b \geq 0\)ですから,\(\sqrt{a}>b\)の両辺を2乗することができて,例えば
\begin{align*}
&\sqrt{a}>b \land b \geq 0 \text{(ア)}\Longrightarrow a > b^2\tag{1}
\end{align*}
のように必要条件が得られます.次にこの\((1)\)における十分性\((\Leftarrow)\)を考えてみましょう.両辺が正ですから,\(\sqrt{\quad}\)をとることができますが,
\[a > b^2 \Longrightarrow \sqrt{a} > \sqrt{b^2} \Longrightarrow \sqrt{a} > |b|\]
となり(ア)に戻れません(\(b\)の正負がわからない).そこで,\((1)\)において(ア)の必要条件をもう少し絞り出しておきましょう.
\[\sqrt{a} > b\land b \geq 0 \text{(ア)}\Longrightarrow a > b^2 \land b \geq 0\]
そして十分性を確認してみます.
\begin{align*}
a > b^2 \land b \geq 0\Longrightarrow &\sqrt{a} > \sqrt{b^2} \land b\geq 0 \\
\Longrightarrow &\sqrt{a} > |b| \land b \geq 0 \\
\Longrightarrow &\sqrt{a} > b \land b \geq 0
\end{align*}
となりこれなら(ア)に戻れます.これで必要十分(同値)であることが分かりました.したがって\((1)\)の論理式は,
\[\sqrt{a} > b \land b \geq 0 \text{(ア)}\Longleftrightarrow a > b^2 \land b \geq 0 \tag{1′}\]
と書きかえれば同値になることが分かりました.

(次に(イ)について)
必要性\(\Rightarrow\)から見てみます.ここでは例えば明らかな必要性
\[(\sqrt{a} > b \land b < 0) \text{(イ)} \Longrightarrow b < 0 \tag{2}\] を考えてみます.逆(十分性)はどうか? \[b < 0 \Longrightarrow (\sqrt{a} > b \land b < 0) \text{(イ)}\]が言えるか?…残念ながら言えません.なぜなら\(a\)は\(\sqrt{\quad}\)の中にあるのだから正でなくてはなりませんが,しかし仮定には\(a\)の正負についての言及がないからです.このことを踏まえて\((2)\)で(イ)の必要条件を適切に絞り出しておきます.\(\sqrt{\quad}\)の‘中身’は正であることに着目して, \[(\sqrt{a} > b \land b < 0) \text{(イ)} \Longrightarrow a \geq 0 \land b < 0\] さてこれならどうでしょうか?逆(十分性)を見てみると \[a \geq 0 \land b < 0 \Longrightarrow (\sqrt{a} > b \land b < 0) \text{(イ)}\] は確かに言えます.したがって,\((2)\)の論理式は \[(\sqrt{a} > b \land b < 0) \text{(イ)} \Longleftrightarrow a \geq 0 \land b < 0\tag{2'}\] と書けば同値であることがわかりました. \((1'),(2')\)により,

\[\sqrt{a}>b \Longleftrightarrow (a > b^2 \land b \geq 0) \lor (a \geq 0 \land b < 0)\]

と同値変形できることが分かりました.

ちなみにもし,\(b \geq 0\)という条件を‘大前提’として奉れば,当然
\[\sqrt{a}>b \Longleftrightarrow a > b^2 \land b \geq 0\]
と書けます.

© 2024 佐々木数学塾, All rights reserved.